Sie sind auf Seite 1von 38

Chapter 18 Verbal Reasoning Learning Objective The objective of this chapter is to expose the student to different kinds of questions

that are asked in exams. Care has been taken to include questions from CAT and other exams. Students should not get surprised to see a new type of question and this chapter covers different types of questions. Through this chapter a student should: Understand what is Verbal Reasoning Learn to attempt different kinds of questions Do exercises that have different kinds of questions.

Verbal Reasoning is usually asked in management entrance tests. It used to be asked earlier in CAT but in recent years it has decreased. However, it is quite possible that some of the questions may make a come-back in the coming years. In all other exams, such as FMS, XAT, IIFT, JMET, IRMA, MAT and others, Verbal Reasoning is an integral part. Students must be aware of the different types of questions and also how to attempt them. In this chapter, we cover the commonly asked types of questions, such as: 1. Analogies 2. Classifying statements into Fact, Inference, Judgement 3. Logical Inference from given statements 4. Analyzing strengths of weakness of given arguments 5. Identifying implicit arguments 6. Analysing which courses of action follow from given premises 7. Judging whether inferences are true or false 8. Cause and effect relationships For each of the above, we have given strategies for attempting the questions. Adequate numbers of questions have also been provided for each category so that the student can practice and develop confidence in this crucial area. In the first section, all the different types of questions have been explained and in the second part, exercises are provided. Description of different types of questions Type 1. Analogies In this type of question, two words are given. The student has to find out the relationship between the two words, and then tick from the answer choices the pair of words having the same relationship. Clearly, the student should have a good vocabulary to do these questions. Sometimes, as in FMS and XAT, the words are quite difficult. Sometimes analogies may consist of three words, which make the questions more difficult. Such questions have been asked in IIT-JMET exam, and have been included in the exercise. The strategy for both types of questions is that the student should try to make sentences between the words given. Then substitute the given answer choices to see if the same sentence holds. The following examples will make the strategy clear. Example 18.1 DIRECTIONS: Write the letter of the pair of words related to each other in the same way as the word in tile capitalized pair. [IIFT 2005] 1. POLE VAULTING : DECATHLON 1. goalpost : football 2. arena : bullfight 3. bull pen : baseball

4. calf-roping : rodeo

5. rink : ice hockey 3. spendthrift : foresight

2. NONAGENARIAN : LONGEVITY 1. truant : attendance 2. zealot: enthusiasm 4. chatterbox : silence 5. child : experience 3. HYPOSENSITIZE : SENSITIVE 1. enlighten : knowledgeable 4. demote : important 4. ARSON : FELONY 1. treason : misdemeanor 4. spy : espionage 2. congratulate : proud 5. educate : independent 2. white lie : perjury 5. hydrogen : element

3. liberate : free

3. contraband : arrest

How to attempt: Look at the given words. What is the relationship between them? In the first question, we see that Pole-vaulting is an event in decathlon. Now apply the same logic to the answer choices. We see that none of the choices match the logic of event except the fourth choice: calf roping in an event in rodeo. In the second question, we see that nonagenarian is a symbol or extreme example of longevity. Applying the same logic to the answer choices, we can get the answer that zealot is an extreme form of enthusiasm. In the third question, the two words are equivalent words. Our job is to see equivalent pairs of words in the answer choices. We see that enlighten and knowledgeable mean the same thing, so this should be the answer. In the fourth question, we can form the sentence: Arson is a kind of felony. Can we get the same sentence from the choices? We see that we can make the same sentence in the fifth choice: Hydrogen is a type of element. What we have learnt: For attempting analogies, develop a very good vocabulary. In the above questions, we come across some difficult words: rodeo, nonagenarian, felony, and so on. Then start looking for relationships between the words given. Look for the same relationship in the answer choices. Type 2. Classifying statements into Fact, Inference, Judgement In this type of question, some statements are given, and the student has to figure out which of them is fact, inference or judgement. Though the questions are not difficult, an exposure to statements and their analysis is required. This kind of question staged a come-back in CAT after many years, but is commonly asked in other management entrance exams. DIRECTIONS: Each question has a set of four sequentially ordered statements. Each statement can be classified as one of the following: [CAT 2006] Facts, which deal with pieces of information that one has heard, seen or read, and which are open to discovery or verification (the answer option indicates such a statement with an 'F'). Inferences, which are conclusions drawn about the unknown, on the basis of the known (the answer option indicates such a statement with an 'I'). Judgements, which are opinions that imply approval or disapproval of persons, objects, situations and occurrences in the past, the present or the future (the answer option indicates such a statement with a 'J'). Select the answer option that best describes the set of four statements. Example 18.2

1. So much of our day-to-day focus seems to be on getting things done, trudging our way through the tasks of living -- it can feel like a treadmill that gets you nowhere; where is the childlike joy? 2. We are not doing the things that make us happy; that which brings us joy; the things that we cannot wait to do because we enjoy them so much. 3. This is the stuff that joyful living is made of -- identifying your calling and committing yourself wholeheartedly to it. 4. When this happens, each moment becomes a celebration of you; there is a rush of energy that comes with feeling completely immersed in doing what you love most. 1. IIIJ 2. IFIJ 3..JFJJ 4. JJJJ 5. JFII Example 18.3 1. Inequitable distribution of all kinds of resources is certainly one of the strongest and most sinister sources of conflict. 2. Even without war, we know that conflicts continue to trouble us - they only change in character. 3. Extensive disarmament is the only insurance for our future; imagine the amount of resources that can be released and redeployed. 4. The economies of the industrialized western world derive 20% of their income from the sale of all kinds of arms. 1. IJJI 2. JIJF 3. IIJF 4. JIIF 5. IJIF How to attempt: 18.2: Note that each of the given statements are personal opinions. The author is lamenting the fact that we have lost our childhood joys. This, of course, can vary from person to person. Further, the authors value judgements seem to be included in the statements. We can thus see that all the statements mention some element of judgment, hence (4). 18.3: Statement I mentions strongest and most sinister which implies judgment. Statement II is an inference as it draws conclusion from war. Statement III is a judgment as it mentions insurance for our future. Type 3. Logical Inference from given statements or assumptions on which they are based A. IDENTIFYING ASSUMPTIONS Example 18.4 DIRECTIONS: Each question has a statement followed by two assumptions. Find the implicit assumptions. Statement: Assumptions: I. II. Economic development leads to educational development in a country. Educational development leads to economic development in a country. 1. Both I and II are implicit 3. Only I is implicit 2. Only II is implicit 4. Neither I nor II is implicit A good system of education in a country is the flower of economic development; it is also its seed.

How to attempt: Analyse the statement and the given assumptions. Ask yourself the question, why is the statement being made? In this case, why doe we say that: A good system of education in a country is the flower of economic development? The answer is that the author has tried to link two variables, education and economic development. Now look at the given assumptions. We find that in both the statements the variables are linked. So we can say that both I and II are implicit.

B. VENN DIAGRAMS In this type of logical inference, some statements will be given which have to be solved by using Set Theory or by making Venn Diagrams. Example 18.5 DIRECTIONS: In each of these questions a few statements are followed by four conclusions numbered I, II, III and IV. Consider the given statements to be true even if they seem to be at variance with commonly known facts. Read all the conclusion(s) and then decide which of the given conclusion(s) logically follow(s) from the given statements. Statements: A. All mirrors are phones. B. Some phones are gadgets. C. All gadgets are mirrors. Conclusions: I. Some gadgets are phones. II. Some gadgets are mirrors. III. Some gadgets are not mirrors. IV. Some mirrors are phones. 1. Only I and II follow 3. Only II and III follow

2. Only I, II & IV follow 4. Either II or IV follow

How to attempt: From the given statements we make a Venn diagram that helps us visualize the problem: A. All mirrors are phones. B. Some phones are gadgets. C. All gadgets are mirrors.

Mirrors Gadgets

Phones

From (A) we see that Mirrors should be a subset of Phones, while Gadgets should intersect Phones. But from C we see that all gadgets should be a subset of Mirrors as well. So the only way to place them is inside the set for mirrors. Now read the conclusions. We see that Gadgets are part of Phones, so I is correct. Similarly, Gadgets are part of mirrors so we can definitely say that some gadgets are mirrors. However, III is wrong, since no gadgets are outside mirrors. IV is also correct. So the answer is (2). Note that without making the diagram, it would not be possible to do the sum correctly. C. LOGICALLY CONSISTENT STATEMENTS In this type of question, several statements are given and the student has to pick out two or more statements which are logically consistent. Example 18.6: DIRECTIONS: Each question has a main statement, followed by four statements labelled A, B, C and D. Choose the ordered pair of statements where the first statement implies the second, and the two statements are logically consistent with the main statement. Statement: Every player will become a champ. A. Rajesh is a player. B. Rajesh will become a champ.

C. Rajesh is not a player. 1. DA 2. AD

D. Rajesh will not become a champ. 3. CD 4. DC

How to attempt: The statement pairs given in the answer choices must be read. Reading AD, can we come to the conclusion of the main statement? Since every player will become a champ, it follows that if one is not a champ, one is not a player. Note the difference between CD and DC. CD implies that if one is not a player, one cannot become a champ. But there could be other ways of becoming one. Since player is a subset of champ, the reverse is not true. Hence the answer is DC. Type 4. Analyzing strengths or weakness of given arguments Questions on analyzing strengths or weakness of arguments may take various forms. A common form of the question is given below: Example 18.7 DIRECTIONS: The question is followed by two arguments numbered I and II. Mark answer as 1. if only argument I is strong 2. if either I or II is strong 3. if only argument II is strong 4. if neither I nor II is strong Statement: Should the fees of all the private professional colleges be made equal to those of the government professional colleges?

Arguments : I. No, the private colleges need additional funds to maintain quality of education. II. Yes, otherwise a large number of meritorious students will not be able to study in these colleges as they charge exorbitantly high fees. How to attempt: Reading the statements we have to see whether the question is logically answered by the arguments. We see that the first argument is not strong, as the same argument can be made for government colleges. The second argument, however, is strong as it says that students may not be able to study there. Type 5. Identifying implicit arguments This type of question is similar to one above, except that here we have to see which statement is implicit in the argument. Example 18.8 DIRECTIONS: A statement is followed by two assumptions numbered I and II. An assumption is something supposed or taken for granted. Consider the statement and the following assumptions. Mark answer as 1. if only assumption I is implicit 2. if either assumption I or II is implicit 3. if only assumption II is implicit 4. if neither assumption I nor II is implicit Statement: The Mohan Cooperative Housing Society has put up a notice at its gate that sales persons are not allowed inside the society.

I.

Assumptions: All the sales persons will stay away from the Mohan Cooperative Housing Society.

II. The security guard posted at the gate may be able to stop the sales persons entering the society. How to attempt: Reading the statement we have to find out why such a board has been placed. I is not implicit, because sales people will still keep coming. But II is a reason, since such persons can be stopped by the guard. Type 6. Analysing which courses of action follow from given premises In this type of question, the student has to see which course of action can be logically taken to tackle a given situation. Example 18.9 DIRECTIONS: A statement is followed by two courses of action numbered I and II. Assume everything in the statement to be true, and decide which of the suggested course(s) of action logically follow(s) for pursuing. Mark answer as 1. if only I follows 2. if either I or II follows 3. if only II follows 4. if neither I nor II follows Statement: There has been significant drop in the water level of all the lakes supplying water to the city.

Course of action: I. The water supply authority should impose a partial cut in supply to tackle the situation. II. The government should appeal to all the residents through mass media for minimal use of water. How to attempt: Since the water levels is low, clearly, we must conserve water. Reading the statements we can see that both are aiming to conserve water. Hence both courses of action would follow. Type 7. Cause and effect relationships A. CAUSE AND EFFECT In this type of question, two events are given and the student has to identify which one is a cause and which one is the effect. Examples DIRECTIONS: Given below are pairs of events I and II. Read both the events and decide the relationship. Assume that the information given is true in deciding the answer. Mark answer as 1. if I is an effect but II is not its immediate and principal cause. 2. if I is the immediate and principal cause and II is its effect. 3. if I is an effect and II is its immediate and principal cause. 4. if II is an effect but I is not its immediate and principal cause. Example 18.10. Event (I): The price of gold has gone up in the local market. Event (II): Indians have won several prizes in designing gold ornaments. Example 18.11.

Event (I): Today, the prime ministers of countries P and Q have decided to take steps to improve bilateral relations. Event (II): Next week a committee of foreign ministers and senior officers of country P and Q will work out further steps to improve the relationship. How to attempt: Read the statements in example 18.15 carefully. The fact that price of gold has gone up in the local market is an effect of some causes, which are not stated. It is certainly not a cause. But on reading the second statement we find that the events are not related. So we say that I is an effect but II is not its immediate and principal cause. In Example 18.16, we see that (II) is an action contemplated for the future. Is (I) the cause? We find that the events are related, and that II is a result of I. Hence we can say that I is the immediate and principal cause and II is its effect. B. SITUATION AND OUTCOME In this type of question, a situation is given and several questions are framed on it to assess whether the outcome follows or not. Example 18.12: DIRECTIONS: A situation and the outcome are presented. Four statements follow thereafter. Each statement is to be separately evaluated in relation to the situation and outcome. Mark answer as 1. if the statement is deducible from the situation, the outcome or both together. 2. if the statement presents a possible adequate explanation of the outcome. 3. if the statement is inconsistent with, or contradicts, the situation, the outcome, or both together. 4. if the statement does not support a possible explanation of the outcome. Situation: Abhijit Roy is training for a national swimming meet. His event is 800 m freestyle. In winning the last five races, his time has never exceeded 8 minutes. His practice performances, in which he studiously attempts to duplicate all actual racing conditions, have been better. He is a strong favourite among local sportswriters to win the meet, and his coach Prabhu Dayal predicts that he will win in record time. Speed Swimming Gear, in the hope of capitalising on his upcoming victory, has persuaded Roy to be photographed with their goggles, which he always wears in competition. Outcome: Roy clocks his worst time ever and finishes fourth. 1. 2. The Speed Swimming Gear company was confident that Roy would win the race. Roy's coach had cautioned him not to expect to do as well as he had in past races.

How to attempt: First read the situation and understand it. We see that though the photographs are taken, he does not win. Why were the photographs taken? The company was confident of his win, so in the first question, the statement is quite correct. So we can say that the statement is deducible from the situation. The second statement contradicts the facts given, because we see that his coach Prabhu Dayal predicts that he will win in record time. So we can say that for the second statement, the statement is inconsistent with, or contradicts, the situation, the outcome. What we have learnt: Argumentative questions can take many forms. The logic of attempting the questions remains same. The student should do all kinds of questions but also keep in mind that if a new question is included in the exam, it is not a cause of distress. Argumentative questions can ususllay be done quite easily.

LOD Exercises These exercises are based on different types of questions LOD-1 Exercise 18.1 (Classifying statements into fact, inference, judgement) Directions: From the alternatives, choose the one which correctly classifies the four sentences as F: Fact: if it relates to a known matter of direct observation, or an existing reality or something known to be true. J: Judgment: if it is an opinion or an estimate or anticipation of common sense or intention, I: Inference: if it is a logical conclusion or deduction of something, based on the knowledge of facts. 1. A. The final award of the Cauvery Water Disputes Tribunal has come 16 years after the Tribunal was set up. B. Now a protracted and acrimonious river water dispute is at a qualitatively new stage with a water-sharing formula awaiting implementation. C. It has been a long wait for the verdict, and not all the claimants are happy with what they have got. D. Yet, the award is a step in the right direction. 1. FIIJ 2. FJJI 3. JFFJ 4. FIJJ 5. IFFI 2. A. The Nithari serial killings confirm that we have ceased to be a nation that cares for the physically weak and socially underprivileged. B. Many of them came from the usually neglected sections of the community such as Dalits. C. The police were hardly stirred by the loss that these hapless villagers had suffered and did nothing to locate the children who had vanished over a period of time until the media made a noise about it. D. This showed how callous the police is. 1. IFFI 2. FIIJ 3. JJJI 4. FIJJ 5. IJJI 3. A. The government's efforts to alter the rural environment is too well known to recount. B. Some amount of rebuilding was done, and one sees evidence of this in the flyovers, the new buildings, malls, and residential complexes. C. A showpiece is the information technology complex in Salt Lake City, as every resident of Kolkata will proudly tell you. D. There are other showpieces as well. 1. IFIJ 2. JIIF 3. FIIJ 4. JIFF 5. IFJF 4. A. In the late 18th century, Bihar came under the notorious Permanent Settlement Act, which by introducing an intermediary layer of parasitic elite in the countryside, imposed a heavy burden on the peasantry. B. Although Bihar was one of the first States to undertake land reforms, the landed gentry thwarted any meaningful reform. C. Alakh Sharma observes that "gross violations" of land ceiling laws in the State thwarted progress in agriculture. D. Because of this, naxalism is raising its head in the state. 1. IJFI 2. IFJI 3. JIII 4. JIFI 5. FIJI

5. A. Known to his friends as `tanker', after the kind of truck he drove for the Jammu and Kashmir Police (JKP), Paddar is thought to hold the clues to a horrific series of cold-blooded murders carried out by counter-terrorism forces in Jammu and Kashmir. B. Acting on behalf of a group of rogue police and Army officers, Paddar is alleged to have kidnapped residents of the Kokernag area of southern Kashmir. C. He then, investigators believe, handed them over to the military and police units who murdered the villagers and passed off their bodies as those of unidentified terrorists. D. The police are in a fix because of the revelations. 1. IFII 2. JIFI 3. FFII 4. IIFI 5. IFIF 6. A. Indian public sector majors Bharat Sanchar Nigam Limited (BSNL) and Mahanagar Telephone Nigam Limited (MTNL) have promised to deliver a New Year gift to those subscribing to their Internet services. B. Bandwidth on entry-level ADSL (asymmetric digital subscriber line) broadband connections provided through telephone lines are to be raised from 256 kbps to 2 mbps and will be a boon to the customer. C. If implemented successfully, the scheme would enhance connectivity in thousands of homes and offices to levels where users can access text, data, voice and video with ease. D. This trend is bound to accelerate, as private Internet service providers (ISPs) would be forced to match the public sector offer. 1. IFIJ 2. JIIF 3. FIIJ 4. JIFI 5. FJIJ 7. A. A survey conducted jointly by the Internet and Mobile Association of India and IMRB International put the number of Internet users in urban India in September 2006 at 37 million. B. That is indeed large when compared to the circulation of English language newspapers in the country, placed at 27 million. C. And the figure could increase rapidly if moves such as the one initiated by BSNL and MTNL expand broadband connectivity in the Year of Broadband. D. Perhaps it is time newspapers in India began reworking their strategies for the Web. 1. IFIJ 2. JIII 3. FIIJ 4. FJIJ 5. IFJI 8. A. We have things like the Planning Commission, which I suppose, spends its day planning things. B. But then there is nothing much to plan either, because everything is being done by the private companies these days. C. We also have a ministry to look after children and women welfare but none of the women or children are any better for it. D. As a result, we have thousands of bureaucrats who have not much to do except become what may be termed as the jerk police. 1. IIJI 2. JIIJ 3. FIIF 4. JIIF 5. IJJI 9. A. When it first became part of the English vocabulary in the early 1990s, `globalisation' was supposed to be the wave of the future. B. Fifteen years ago, the writings of globalist thinkers such as Kenichi Ohmae and Robert Reich celebrated the emergence of the so-called borderless world. C. The process by which relatively autonomous national economies become functionally integrated into one global economy was touted as "irreversible". D. A fresh thinking is emerging about the issue now. 1. IFJI 2. FFJI 3. FIIJ 4. JIFJ 5. IJFF 10. A. Before The God Delusion was published, Dawkins wrote about something called Gerin oil that was poisoning human society. B. Gerin oil (or Geriniol, to give it its scientific name) is a powerful drug that acts directly on the central nervous system to produce a range of characteristic symptoms, often of an antisocial or self-damaging nature.

C. If administered chronically in childhood, Gerin oil can permanently modify the brain to produce adult disorders, including dangerous delusions that have proved very hard to treat. D. The four doomed flights of September 11 were, in a very real sense, Gerin oil trips: all 19 of the hijackers were high on the drug at the time. 1. JIFI 2. FJII 3. FFIJ 4. FFII 5. JFII 11. A. The capacity to absorb investment is not a stand-alone intrinsic factor of the scientific community; it is actually a function of several factors that depend on the systems and mechanisms that are in (or not in) place for the administration of S&T in the country. B. Investing 2 per cent of the GDP in S&T is indeed a desirable objective if we wish to become a developed nation. C. The President urged that there be a focussed action plan to realise this progressively and suggested the constitution of a joint team comprising members from all the scientific departments of the government to work out the growth plan in an integrated way. D. Significantly, he recommended allocating 0.5 per cent of the GDP for basic research as against the present level of around 0.2 per cent. 1. FFFI 2. FIIJ 3. FJFF 4. FIFF 5. IJFF 12. A. The passage of the Scheduled Tribes and Other Traditional Forest Dwellers (Recognition of Forest Rights) Bill, 2006, is an important step in the struggle to reverse the historical marginalisation of the tribal people of India. B. The Scheduled Tribes, constituting about 8.4 per cent of the population, have been denied access to benefits from land and forests by both the medieval and the modern state. C. Thus, the displacement of tribal people into forests was not a colonial phenomenon alone, but British imperialism accentuated it by setting up state monopoly over forests. D. As a consequence, a centralised and often autocratic forest management came into force in India. 1. JIFI 2. JIIF 3. FIIJ 4. FIII 5. JIII 13. A. In their ongoing study of wild balsams, researchers of the Edhkwehlynawd Botanical Refuge of Udhagamandalam noticed that some of the native species had become rare. B. It took them three years to locate Impatiens denisonii; they made three annual field trips during August-September, when the balsams are in bloom, and reported the sighting of the species on the third such visit. C. It is likely that this was the first scientific collection of the species since British naturalist Richard Henry Beddome first documented it in 1862. D. The study was the most authoritative text on the subject and was prescribed in universities. 1. IFIJ 2. IJIF 3. IIJF 4. JIIJ 5. IFJI 14. A. Watching Tony Blair's awkward demeanour alongside George Bush at the White House, it was striking just how American a country Britain has become. B. It has long been a cliche that the UK is the 51st state of the union, but it has never seemed a more appropriate description. C. Indeed, there is a case for saying that after effectively living in sin for so long, it's time to make the "special relationship" legitimate. D. Britain is never going to join the USA, but if it did the prime minister would probably have a lot more clout on the other side of the Atlantic than he does now. 1. FIJJ 2. JIIJ 3. IJJJ 4. JIFJ 5. IJJF 15. A. The problem is that in exercising one precious right, we often extinguish that right in others -- those who constitute what is called the unorganised sector of society. B. When a political party or a trade union, or a group of trade unions, decides to call a bandh and shut down a State for a day, lakhs of people who are in the unorganised sector, lose their earnings for that day.

C. Most of them depend on their daily earnings to manage their households and have to do without the means to feed their families. D. More often than not, bandhs have no effect on public awareness of the reasons they were organised for, except in a vague way. 1. IIIJ 2. JIII 3. IIIF 4. JIIF 5. IIFF 16. A. A hard landing for the Indian economy is quite apparent from the super-charged property market. B. Property prices are rising at an awesome rate of 10-30% each quarter and are beyond the reach of most prospective buyers. C. In the suburbs of Mumbai one can see block after block of empty buildings amid the new construction projects, yet private banks enthusiastically lend money on the premise that the economy will continue to grow at a high rate. D. With the derivatives market in a nascent stage there is little that they can do to offload these risks from their balance sheets. 1. IFII 2. IFJI 3. FIIJ 4. IJFI 5. IIFI 17. A. The peace process reflects the acceptance by General Pervez Musharraf, Pakistan's leader, of what has long been obvious: big India will never concede an inch of Kashmir, whatever little Pakistan does. B. General Musharraf has therefore accepted something very close to what India demands. C. That General Musharraf could contemplate such an agreement is humiliating to many Pakistanis. D. Yet he believes that, for the benefits of peace, they will swallow this humiliationas long as India accepts the small compromises he has suggested and provided peace comes quickly. 1. IIFI 2. IIJF 3. FIJI 4. FJIF 5. JIFF 18. A. Both India and China have large populations, low incomes and rapidly rising GDP, yet the composition of their growth has been quite different. B. A recent paper by Barry Bosworth and Susan Collins, of the Brookings Institution in Washington, DC, explores the sources of expansion in both countries C. They break down total GDP growth into increases in inputs of labour and capital, and gains in TFP. D. Employment increased faster in India than in China, but this was more than offset by a much slower rise in output per worker: only 4.6% a year, compared with 8.5% in China. 1. JIFF 2. JFFJ 3. FJII 4. IFFI 5. IIFF 19. A. The late Srila Prabhupada, of the Hare Krishna movement, explained that God created "the 8,400,000 species of life from the very beginning," in order to establish multiple tiers of reincarnation for rising souls. B. Although souls ascend, the species themselves don't change, he insisted. C. He dismissed "Darwin's nonsensical theory." D. Other people too, not just scriptural literalists, remain unpersuaded about evolution. 1. IFIJ 2. JIIF 3. JJJI 4. JIII 5. FFFI 20. A. The workers took eggs containing Drosophila embryos in various stages of early development, and soaked them in a solution of DNA. B. This material (deoxyribonucleic acid) occurs in the genes of the chromosomes, and is now thought to contain, in the famous genetic code. C. In these experiments, the DNA was obtained from other flies differing in several ways from those providing the eggs. D. The differences involved ten or eleven genes at various places on the chromosomes, and affected such characteristics as the structure and colour of bristles, wing veins, wing structure, and eye colour. 1. FIFF 2. JIJJ 3. FIIF 4. JIFF 5. IFFF

21. A. Post the Attari Express blast, senior government officials say "the entire context of the joint mechanism has changed. B. India has been put in the predicament of now having to explain to Pakistan how its citizens lost their lives in India. C. In the public battle for hearts and minds, India would now have to provide a progress report on the blast investigations, thereby allowing Pakistan to exploit a huge window of unexpected opportunity. D. The Pakistan National Assembly in a unanimously adopted government-sponsored resolution called for the formation of a joint Pakistan-India investigation team for the probe. 1. FIII 2. FJIJ 3. FJIF 4. FFII 5. JFIF 22. A. Given this mindset among policymakers, big-ticket reforms may be out in the Budget. B. For example, the FM could have hinted at allowing FDI in retail. C. But after the letter Congress president Sonia Gandhi wrote to the PM voicing her concerns on the subject, this is a no-mover. D. Although Chidambaram has taken a few steps to dilute government stake in PSUs, he is unlikely to use the word 'disinvestment' this year. 1. IFIJ 2. IJIJ 3. JJJI 4. JIII 5. IJFI 23. A. If our security forces are doing a stellar job, as some experts believe, then there is no reason for them to be so secretive about their successes. B. If it is true that for every hideous act that succeeds there are ten that fail because our men in khaki have destroyed terrorist modules before they commit their evil deeds, then let us have names, faces, interviews on television and details of how the police caught who they did. C. We need to learn from the West. D. Within days of 9/11 the whole world knew how many hijackers there were, where their money came from, which flying schools they were attending and what their nationalities were. 1. IIJF 2. JIIF 3. IIIF 4. JIFF 5. IIFJ 24. A. Mr Quattrocchi was detained at an airport in Argentina following a red corner alert issued by Interpol. B. Later its spokesmen explained away the silence by informing media that they wanted to make sure about the identity of the person detained! C. They admitted that the detention was a complete surprise and attributed it to chance. D. It is clear that the CBI wanted to oblige Mrs Sonia Gandhi and to save her from the embarrassment. 1. FIJF 2. JFFJ 3. JIIF 4. IFFI 5. JIFJ 25. A. What do you expect of ordinary people if MLAs of a national political party like the Trinamul Congress, think nothing of breaking tables and chairs in the legislative assembly? B. They did this because their leader, Mamata Bannerji, was allegedly denied permission to protest in the streets. C. If this is not primitive politics what is? D. Mobs are faceless, so the Dalit vandals who burned buses and the Deccan Queen will probably get away with what they did, but Ms Bannerji must be made to pay for the damage her MLAs did. 1. FIFF 2. JIIF 3. JIJJ 4. JIFF 5. FIJJ 26. A. The prime need is to create an instrument for governance. B. That implies a cohesive and stable government. C. The era of coalitions has made it painfully clear that makeshift alliances are unsuited for purposive governance. D. Some years ago Narasimha Rao was unhappy with the state of affairs. 1. JIIF 2. FIIF 3. FIIJ 4. JIFF 5. IIFJ

27. A. The I&B ministry, in its endless efforts to be the real Big Brother, reminds us that it is a leftover of bad times. B. It had a role in the days when Doordarshan was our only television channel, when film-makers were so poor they needed government help to make dreary, un-entertaining art films that no sane financier would pay for and, when it was possible, to pass government propaganda as news. C. The ministry is now obsolete and the quicker it is abolished the better. D. I also believe that its time we abolished the union ministry of human resource development and used this change to put Arjun Singh out to pasture. 1. IIJF 2. JIIF 3. FFJJ 4. JJFF 5. IIJJ 28. A. Indian farmers cannot have big farms (except illegally) because of land ceiling laws that should have been abolished years ago. B. They benefit nobody and keep the prices of farm land artificially low. C. Then, along comes some state government with a big industrial project in its bag and acquires the land at this artificially low price. D. The value of the land shoots up when a Ratan Tata puts in the roads, power and other infrastructure that comes with industrialisation, and the farmer feels cheated. 1. FIJF 2. IJFI 3. JIIF 4. JJFF 5. IIFI 29. A. Incredible India! is our slogan to woo foreign visitors to our old and decrepit land. B. And, India is truly incredible but not always in the best sense of that word. C. Just as the word incredible has an ambiguity so does our approach to tourism. D. So, on the one hand the Government of India enthusiastically promotes its Incredible India! campaign to lure visitors from far and wide, and on the other hand once they get here we harass them in varied, very Indian ways. 1. IIIJ 2. JJJJ 3. IIJJ 4. JJFF 5. JJJI 30. A. A campaign to play down the culpability of minority communities and to direct the attack against the administration and the majority community is nothing new in our history since 1947. B. This has always happened after every communal riot. C. Whenever some Muslims take the law into their own hands, it is always the police which is criticised for acting against them. D. The secular elite rarely criticises the Muslims, who violated the law in the first place, and rarely calls for action against them. 1. JJJJ 2. JIIF 3. IIIF 4. JIFF 5. IIFJ Exercise 18.2 Directions Q 1 to 5: Select the pair of sentences that relate logically with the given statement. [CAT 1997] 1. Either Sam is ill; or he is drunk. A. Sam is ill. B. Sam is not ill. C. Sam is drunk. a) AB b) DA c) AC d) CD 2. Whenever Ram hears of a tragedy, he loses sleep. A. Ram heard of a tragedy. B. Ram did not hear of a tragedy. D. Ram did not lose sleep. a) CA b) BD c) DB d) AD D. Sam is not drunk.

C. Ram lost sleep.

3. Either the train is late; or it has derailed. a. The train is late. b. The train is not late. c. The train is derailed. d. The train is not derailed. a) AB b) DB c) CA d) BC

4. When I read a horror story I have a nightmare. a. I read a story. b. I did not hear a horror story. c. I did not have a nightmare a nightmare. a) CB b) AD c) BC d) AC 5. When I eat berries I get rashes. a. I ate berries. b. I did not get rashes. c. I did not eat berries. d. I got rashes. a) DA b) BC c) CB d) AD

d. I had

DIRECTIONS for questions 6 to 10: In each question below are three statements followed by three conclusions numbered I, Hand III. You have to take the three given statements to be true even if they seem to be at variance from commonly known facts and then decide which of the given conclusions logically follows from the three given statements disregarding commonly known facts. Then decide which of the answers 1, 2, 3, 4 and 5 is the correct answer and indicate it on the answer sheet. [CET 2006] 6. Statements: Conclusions: Some chairs are tables. Some tables are drawers. All drawers are shelves. I. Some shelves are tables. II. Some drawers are chairs. III. Some shelves are drawers. 1. Only I and III follow 2. Only I and either II or III follow 3. Only II and either I or III follow 4. All I, II & III follow 5. None of these Statements: Conclusions: All trees are flowers. Some flowers are leaves. No leaf is bud. I. No bud is a flower. II. Some buds are flowers. III. Some leaves are trees. 1. Only II & III follow 2. Only III follows 3. Only either I or II follows 4. Either I or II and III follow 5. None of these Statements: Conclusions: All stones are rocks. Some rocks are bricks. Some bricks are cement. I. Some cements are rocks. II. Some cements are stones. III. No cement is stone. 1. Only I and either II or III follow 2. Only either II or III follows 3. Only I & II follow 4. All follow 5. None of these

7.

8.

9. Statements: All flats are buildings. All buildings are bungalows. All bungalows are apartments. Conclusions: I. Some apartments are flats. II. All flats are bungalows. III. Some bungalows are flats. 1. None follows 2. Only I & II follow 3. Only II & III follow 4. Only I & III follow 5. All I, II & III follow 10. Statements: Some spectacles are lenses. Some lenses are frames. All frames are metals. Conclusions: I. Some lenses are metals. II. Some metals are spectacles. III. Some frames are spectacles. 1. None follows 2. Only I follows 3. Only I and either II or III follow 4. Only I and II follow

5. None of these DIRECTIONS for questions 11 to 15: In making decisions about important questions, it is desirable to be able to distinguish between 'strong' arguments and 'weak' arguments. 'Strong' arguments are those which are both important and directly related to the question. 'Weak' arguments are those which are of minor importance and also may not be directly related to the question or may be related to a trivial aspect of the question. [IRMA 2007] Each question below is followed by two arguments numbered I and II. You have to decide which of the argument is a 'strong' argument and which is a 'weak' argument. Give answer 1 if only argument I is strong. Give answer 2 if only argument II is strong. Give answer 3 if either I or II is strong. Give answer 4 if neither I nor II is strong. Give answer 5 if both I and II are strong. 11. Should the Govt. allow management of all the educational institutions in the country to frame their own policies and make them completely free from Government interference? Arguments: I. No, the management of all the educational institutions are not capable of framing their own policies and this may lead to biased decision. II. Yes, Govt. interference has ruined the very essence of academic freedom the institutions need to enjoy to provide better education. 12. Should there be only one command for all the armed forces in our country? Arguments: I. Yes, this will help effectively coordinating activities of all the armed forces and also will effectively increase the strike capability. II. No, this is not practicable as each navy, air force and army have different strategies and workforce procedures so they need to be under different command. 13. Should all the housing societies in big cities be forced to put water harvesting system in place? Arguments: I. Yes, this is the only way to solve perennial water shortage problem in the big cities. II. No, this is difficult to implement. 14. Should the Govt. allow development of SEZs on the agricultural land? Arguments: I. Yes, this will immensely boost the economy of the rural areas as the returns will be much higher than the agricultural products. II. No, more and more farmers will be dislocated and will lose their livelihood leading to many social imbalances. 15. Should the cultivation of genetically modified cotton varieties be completely banned in India? Arguments: I. Yes, many farmers in the country have suffered immensely and had to abandon their fields after cultivating genetically modified cotton variety. II. No, the farmers can get bumper crop by cultivating genetically modified cotton variety which will significantly improve their financial condition. DIRECTIONS for questions 16 to 20: Below in each question are given two statements A and B. These statements may be either independent causes or may be effects of independent causes or a common cause. One of these statements may be the effect of the other statement. Read both the statements and decide which of the following answer choices correctly depicts the relationship between these two statements. [NMAT 2007] Mark answer 1, if statement A is the cause and statement B is its effect. Mark answer 2, if statement B is the cause and statement A is its effect.

Mark answer 3, if both the statements A and B are independent causes. Mark answer 4, if both the statements A and B are effects of independent causes. Mark answer 5, if both the statements A and B are effects of some common cause. 16. 17. Statements: Statements: set up SEZs. Statements: A. Rise in petrol prices was announced recently. B. Number of cars on the road is increasing constantly. A. A bandh was called on Monday in protest against land acquisition to B. Violence and arson totally disrupted normal life on Monday. 18. A. The health department of the Government of State 'X' ordered all the hospitals to store sufficient stock of medicines to handle the epidemics generally breaking out at the onset of monsoon. B. The health department of the Government of state 'X' campaigned the need for maintaining cleanliness among public particularly those residing in slum areas. A. Rupee has been consistently strengthening for past few months. B. Reserve Bank of India has recently reduced the interest rates. A. An increase in number of cases of depression is being observed these days among the young students. B. Young students have to tackle with more competition than their counterparts faced in the past.

19. 20.

Statements: Statements:

DIRECTIONS for question 21 to 25: Certain Assertions (A) and respective Reasons (R) are given. Choose the correct alternative from the following: [FMS 2005] 1. Both A and R are true and R is the correct explanation of A 2. Both A and R are true but R is NOT the correct explanation of A 3. A is true but R is false 4. Both A and R are false 21. 22. 23. 24. Assertion (A) : Reason (R) : Assertion (A) : Reason (R) : Assertion (A) : Reason (R) : Inside the earth metals are present in molten state. Earth absorbs the sun's rays. There is no vaccine for AIDS. The AIDS virus changes its genetic code. Carbohydrates provide energy to the body. Obesity is caused by excessive intake of carbohydrates. in

Assertion (A) : In India, the judiciary is independent of the executive. Reason (R) : Judiciary favors the government and helps implementation of its plans. Assertion (A) : Reason (R) : India is facing the problem of inflation. We have failed to check the growth of black money.

25.

DIRECTIONS for questions 26 to 30: A statement is given and which is followed by two conclusions I and II. Give the answer on the basis of given statement only: 1. If only conclusion I follows 2. If only conclusion II follows 3. If either I or II follows 4. If neither I nor II follows

26.

Statement Conclusion Conclusion Statement Conclusion Conclusion

I II

: : : :

A neurotic is a non-stupid person who behaves stupidly. Neuroticism and stupidity go hand in hand. Normal persons behave intelligently. Until our country achieves economic equality, political freedom and democracy would be meaningless. Political freedom and democracy go hand in hand. Economic equality leads to real political freedom and Democracy. A corporate general manager asked four managers to either submit their resignations by the next day or face termination of serv ices. Three of them submitted their resignation by that evening. The next day the remaining manager would also resign. The General Manager would terminate his service the next day. The percentage of the national income shared by the top ten percent of household in India is about 35. When an economy grows, concentration of wealth in certain pockets takes place. The national income is unevenly distributed in India. America's defence secretary reiterated that they would continue to supply arms to Pakistan. Pakistan is incapable of manufacturing arms. It would ensure peace in the region. LOD-2 Exercise 18.3 (Analogies)

27.

I II

: : :

28.

Statement

Conclusion Conclusion 29. Statement Conclusion Conclusion 30. Statement Conclusion Conclusion

I II

: : :

I II

: : :

I II

: :

DIRECTIONS for questions 1 to 4: In each of the questions four pairs of numbered words follow a pair of words printed in capital letters. Choose the numbered pair of word that expresses the same relationship as the given pair in capital letters. [FMS 2005] 1. INTERLUDE : REST 1. Vacation : Work 3. Sabbatical : Freedom RACISM : APARTHEID 1. Sexism : Chauvinism 3. Nationalism : Identity SECULAR : SPIRITUAL 1. Amoral : moral 3. Noisome : Hostile SINISTER : HARM 1. Malignant : Humor 3. Ominous : News 2. Retirement : Retreat 4. Intermission : Respite 2. Parochialism : Linguism 4. Communalism : Religion 2. Wet : dank 4. Praise : Embarrassment 2. Horrifying : Trepidation 4. Benevolent : Bountiful

2.

3.

4.

DIRECTIONS for questions 5 and 6: Select the set of words from the given options that BEST expresses a relationship SIMILAR to the set in CAPITALS: [JMET 2007] 5. LIONS : PRIDE : FOREST 1. geese : gaggle : sky 3. fish : shoal : ocean 6. POET : VISION : PROPHET 1. politician : constituency : voter 3. student : school : principal 2. houses : colony : city 4. paper : ream : press 2. killer : violence : terrorist 4. plant : herbivore : food-chain

DIRECTIONS for questions 7 to 15: In each of the following questions, a related pair of words is linked by a colon, followed by four pair of words. Choose the pair, which is most like the relationship expressed in the original pair in capital letters. [FMS 2006] 7. ERROR : INFALLIBLE : : 1. Emotion : Invulnerable 3. Flaw : Impeccable 8. TEARS : LACHRYMOSE : : 1. Words : Verbose 3. Jests : Ironic 9. AUTHENTICITY : APOCRYPHAL : : 1. Wickedness : Nefarious 3. Assertiveness : Dogmatic 10. SERVILITY : GROVEL : : 1. Arrogance : Titter 3. Hypocrisy : Snivel 11. POET : DOGGEREL : : 1. Composer : Symphony 3. Playwright : Soliloquy 12. LIGHT : GLINT : : 1. Colour : Shade 3. Sound : Blare 13. VAINGLORIOUS : SELF-RESPECTING : : 1. Cantankerous : Querulous 3. Punctilious : Careful 14. TOCSIN : DANGER : : 1. Knell : Death 3. Augury : Warning 15. HERO : ACCOLADE : : 1. Defendant : Indictment 3. Mentor : Advice 2. Defect : Intolerable 4. Cure : Irreversible 2. Speeches : Morose 4. Requests : Effusive 2. Artifice : Deceptive 4. Integrity : Hypocritical 2. Modesty : Preen 4. Anger : Fume 2. Painter : Easel 4. Novelist : Potboiler 2. Scent : Whiff 4. Tide : Wave 2. Lascivious : Modest 4. Perfidious : Loyal 2. Clarion : Battle 4. Antitoxin : Cure 2. Laughingstock : Ridicule 4. Clodhopper : Grace

DIRECTIONS for questions 16 to 19: Each question has 4 pairs of words in a set. A set may consist of synonyms or antonyms. [FMS 2006] 16. Mark the pair that does not go with other pairs in the set:

1. Bawdiness : Persiflage 3. Vicarious : Secondhand

2. Platitudinous : Original 4. Ephemeral : Evanescent

17. Mark the pair that does not go with other airs in the set: 1. Enervated : Exhilarated 2. Extravagant : Parsimonious 3. Clich : Bromide 4. Indigent : Opulent 18. Mark the pair that does not go with other pairs in the set: 1. Cadaverous : Gaunt 2. Dolorous : Delightful 3. Obstreperous : Noisy 4. Obsequious : Servile 19. Mark the pair that does not go with other pairs in the set : 1. Catlike : Feline 2. Bearlike : Ursine 3. Horselike : Equine 4. Vulturelike : Vulpine 20. Given below are the names of some phobias (fears). Match the correct group of words with the phobias given in capital letters: CALLOPHOBIA; SCELEROPHOBIA; OCHLOPHOBIA; COPROPHOBIA. 1. Colours; Celebrity; Ocean; Company 2. Company; Burglars; Crowds; Celebrity 3. Beauty; Burglars; Crowds; Faces 4. Beauty; Scenery; Orchestra; Faces Directions Q 21-30: Fill in the blanks by forming analogies. Then tick the appropriate choice from the given answer choices. [SNAP 2006] 21. _____________ is to constitution as prologue is to ______________. 1. Independence 2. Law 3. Preamble A. Eulogy B. Write C. Play 1. 3C 2. 1D 3. 4A 22. ____________ is to horse as bray is to_______________. 1. Drive 2. Hoof 3. Neigh A. Relay B. Pony C. Wagon 1. 1D 2. 1A 3. 3C 23. ___________ is to distance as kilogram is to _____________. 1. Far 2. Meter 3. Europe A. heavy B. ounce C. weight 1. 2A 2. 2B 3. 2C 4. Amendment D. Epilogue 4. 4D 4. Saddle D. Donkey 4.3D 4. Travel D. noise 4. 1A 4. Often D. total 4. 4A 4. D. spillway 4. 1A 4. Conscience

24. ____________ is to never as all is to ___________. 1. Always 2. Usually 3. Seldom A. none B. whole C. every 1. 1A 2. 4D 3. 2B 25. ____________ is to rain as dam is to ____________. 1. Cloud 2. Fog 3. Water Umbrella A. electricity B. flood C. high 1. 3B 2. 1B 3. 4B 26. ____________ is to pacifist as religion is to _____________. 1. War 2. Atlantis 3. Object

A. devout 1. 4C

B. sacred 2. 4B

C. atheist 3. 1C

D. minister 4. 4A 4. Moulting D. farm 4. 2A 4. Colony D. Money 4. 2A 4. Alberta D. Ceylon 4. 3A 4. Full D. many 4. 4D

27. ____________ is to bird as shedding is to ______________. 1. Calling 2. Flying 3. Migrating A. barn B. dog C. hay 1. 4A 2. 4B 3. 2C 28. ____________ is to England as lira is to ______________. 1. London 2. Pound 3. King A. Italy B. mexico C. mandolin 1. 1A 2. 1B 3. 1D 29. ____________ is to Canada as Rangoon is to ____________. 1. Detroit 2. Florida 3. Toronto A. Indonesia B. Burma C. East Pakistan 1. 2B 2. 3B 3. 3D 30. ____________ is to all as part is to _____________. 1. Each 2. Right 3. None A. whole B. Separate C. role 1. 1A 2. 4B 3. 1D

Exercise 18.4 (Argumentative Questions) DIRECTIONS for questions 1 to 4: Each question given below is followed by two arguments numbered I and II. You have to decide which of the arguments is a 'strong' argument and which is a 'weak' argument. Give answer as 1. if only argument I is strong 2. if only argument II is strong 3. if either I or II is strong 4. if neither I nor II is strong 1. Statement : Should India go in for computerisation in industry? Argument : s I. No. Computerisation demands a lot of money. We should not waste money on it. II. Yes. When advanced countries are introducing computers in various areas, how can we afford to lag behind ? Statement : Should personal tax be abolished in India? Argument : s I. Yes. It will motivate people to earn more. II. No. Individuals must learn to share their wealth with other people. Statement : Should high chimneys be installed in industries? Argument : s I. Yes. It reduces pollution at ground level. II. No. It increases pollution in upper atmosphere. Statement : Should judiciary be independent of the executive?

2.

3.

4.

Argument : s I. Yes. This would help curb the unlawful activities of the executive. II. No. The executive would not be able to take bold measures. DIRECTIONS for questions 5 to 8: In each question below there is an inference, which is based on the given passage. Examine each inference separately in the context of the passage and decide upon its degree of truth or falsity. Mark answer as 1. if you think the inference is 'definitely true' 2. if you think the inference is 'probably true' though not definitely true in the light of the facts given 3. if you think the 'data are inadequate' i.e., from the facts given you cannot say whether the inference is likely to be true or false 4. if you think the inference is 'definitely false' PASSAGE The reforms are aimed at having an impact over a wide cross-section of the economy. With improvements and greater efficiency in mobilising and allocating resources, the basis will become stronger for promoting economic growth and development. The reforms aim at increasing efficiency of the financial sector and the range of financial services available within the economy. The introduction of bank capital adequacy and accounting standards, together with improvement in the regulation and supervision of financial institutions and capital market, is aimed at installing greater public confidence, ensuring safety and soundness of the financial system, bringing about greater transparency and accountability in operations and encouraging overall increased resources mobilisation within the economy. 5. Prior to reforms, the banking sector was running inefficiently although resource allocation was efficient. 6. Bank capital adequacy was not in operation before reforms. 7. Public confidence has some connection with resource mobilisation. 8. The economic reforms have aimed only at the financial sector. DIRECTIONS for questions 9 and 10: In each question below are given two statements, followed by four conclusions numbered I, II, III and IV. You have to take everything given in the statements to be true although it may seem at variance with commonly accepted facts. Then decide which of the conclusions follows from the statements. Mark the right answer from (a), (b), (c) and (d). [FMS 2008] 9. Statements: 1. All children are adults. 2. All adults are fat. Conclusions: I. All fat persons are children. II. All children are fat. III. Only some children are fat. IV. Some fat persons are adults. (a) Only I and II follow (b) Only III and IV follow (c) Only II and IV follow (d) Only I and III follow 1. (c) 2. (d) 3. (a) 4. (b) 10. Statements: 1. All stones are marbles. 2. Some marbles are diamonds. Conclusions: I. Some diamonds are stones. II. Some diamonds are not marbles. III. Every diamond is either a marble or a stone. IV. No stone is a diamond (a) Only I and II follow (b) Only II and III follow

(c) Either II or III follows 1. (c)

2. (d)

(d) Either I or IV follow? 3. (a) 4. (b)

DIRECTIONS for questions 11 and 12: In each of the following questions, there is a statement followed by two assumptions I and II. You are to consider each statement and the assumptions that follow and decide which of the assumptions is implicit in the statement. Indicate, your answer as (a) if only I is implicit, (b) if only II is implicit, (c) if neither I nor II is implicit and (d) if both I and II are implicit. [FMS 2008] 11. Statement: We should use detergent to clean objects. Assumptions: (I) Detergents help to dislodge grease and dirt (II) Detergents form more lather. 1. (c) 2. (d) 3. (a)

4. (b)

12. Statement: Every year doctors, scientists and engineers migrate from India to greener pastures. Assumptions: (I) Brain drain has affected India adversely. (II) Better scales and better standards of living act as a bait to lure them. 1. (c) 2. (d) 3. (a) 4. (b) DIRECTIONS for questions 13 to 15: In each question given below there are two statements labelled as Assertion (A) and the other labelled as Reason (R). Of these statements which one of the following is correct? [FMS 2007] (a) Both A and R are true and R is the correct explanation of A. (b) Both A and R are true but R is not the correct explanation of A. (c) A is true but R is false (d) A is false but R is true. 13. Assertion (A): Transformer is useful for stepping up or stepping down voltages. Reason (R): Transformer is a device used in DC circuits. 1. (a) 2. (b) 3. (c) 4. (d)

14. Assertion (A): A beaker filled with water at 4C overflows if the temperature is increased or decreased. Reason (R): Density of water is maximum at 4C. 1. (a) 2. (b) 3. (c) 4. (d)

15. Assertion (A): The emergence of economic globalism does not imply the decline of social ideology. Reason (R): The ideology of socialism believes in universalism and globalism. 1. (a) 2. (b) 3. (c) 4. (d)

DIRECTIONS for question 16 to 20: In each question below are given two statements followed by two conclusions numbered I and II. You have to take the two given statements to be true even if they seem to be at variance from commonly known facts and then decide which of the given conclusions logically follows from the two given statements, disregarding commonly known facts. Read both the statements and give answer (a) if only conclusion I follows. (c) if either I or II follows. (b) if only conclusion II follows. (d) if neither I nor II follows.

16.

Statements: Conclusions: 1. (a)

I: Some artworks are paintings. II: All paintings are master pieces. I: All masterpieces are paintings.II: Some masterpieces are artworks. 2. (b) I: Some men are genius. I: Some brothers are men. 2. (b) I: All pants are skirts. I: Some skirts are pants. 2. (b) I: All planes are tyres. I: No engine is a plane. 2. (b) I: Some cartoons are funny. I: All funny are cartoons. 2. (b) 3. (c) 4. (d)

17.

Statements: Conclusions: 1. (a)

II: No brother is genius. II: Some brothers are not men. 3. (c) II: No shirt is a skirt. II: All shirts are pants. 3. (c) II: All tyres are engines. II: No tyre is a plane. 3. (c) 4. (d) 4. (d) 4. (d)

18.

Statements: Conclusions: 1. (a)

19.

Statements: Conclusions: 1. (a)

20.

Statements: Conclusions: 1. (a)

II: Some cartoons are silly. II: Some silly are cartoons 3. (c) 4. (d)

DIRECTIONS for questions 21 to 23: These questions are presented with three true statements: Fact 1, Fact 2, and Fact 3. Then, you are given three more statements (labeled I, II, and III), and you must determine which of these, if any, is also a fact. [JMET 2007] 21. Fact 1: Manoj said, "Anush and I both went to a movie last night." Fact 2: Anush said. "I was only studying last night." Fact 3: Manoj always tells the truth, but Anush sometimes lies.

If the first three" statements are facts, which of the following statements must also be a fact? I. Anush went to a movie last night. II. Manoj went to a movie last night. III. Anush was studying last night. 1. II only 2. I only 3. I and II only 4. I, II, and III 22. Fact 1: Chairs cost between Rs. 200 to Rs. 2000. Fact 2: Some chairs are made of aluminum. Fact 3: Some chairs are made of plastic.

If the first three statements are facts, which of the following statements must also be a fact? I. Aluminum chairs cost more than plastic chairs. II. Expensive chairs last longer than cheap chairs. III. Plastic chairs costs around Rs. 200 and aluminum chairs costs around Rs 2000. 1. I only 2. II only 3. I and III only 4. None 23. Fact 1: All metros have ring roads. Fact 2: Delhi is a metro. Fact 3: Delhi has a population of more than 5 million.

If the first three statements are facts, which of the following statements must also be a fact? I. Delhi has a ring road. II. All metros have a population more than 5 million. III. All cities with a ring road are metros. 1. I only 2. I and II only 3. I and III only 4. I, II, and II DIRECTIONS for questions 24 to 25: On the basis of the given two facts, determine which of the conclusions marked A, B, C, or D can be most logically drawn. 24. Fact I: Some musicians play Tabla. Fact 2: All the Tabla players need to be trained for at least 10 years. 1. Children of Tabla players may require less than 10 years of training. 2. All the musicians who have trained for at least 10 years are Tabla players. 3. Some of the musicians may have been trained for at least 10 years. 4. All the musicians are Tabla players.

25. Fact 1: At a parking lot a car is parked to the right of a truck and to the left of a van. Fact 2: A jeep is parked to the right of the truck. 1. The car is to the left of the jeep. 2. The jeep is to the right of the van. 3. The jeep is parked between the car and the truck. 4. The truck is to the left of the jeep. Q 26-30: Below is given a passage followed several possible inferences numbered from 66 to 70 which can be drawn from the facts stated in the passage. You have to examine each inference separately in the context of the passage and decide upon- its degree of truth or falsity. [CET 2007] Mark answer (1) if you think the inference is definitely true i:e. it properly fallows from the statement of facts given. Mark answer (2) if you think the inference is probably true though not definitely true in the light of the facts given. Mark answer (3) if you think the data are inadequate i.e. from the facts given you Cannot say whether the inference is likely to be true or false Mark answer (4) if you think the inference is probably false though not definitely false in the light of the facts given and Mark answer (5) if you think the inference is definitely false ie. it cannot possibly be drawn from the facts given or it contradicts the give facts. Now, read, the following passage and the inferences given below them and mark your answers in the answer sheet. The serious accident in which person was run dawn by a car yesterday has again focused attention on the most unsatisfactory state of street lighting. No one expects side roads to be provided with the same standard of lighting, as a man road, but unless the council is prepared to make good its promise as regards road lighting, it will only be a question of time before there are further and perhaps fatal accidents. 26. 27. 28. 29. 30. It seems that the council has promised to improve the state of lighting on side roads. Several accidents have so far taken place because of unsatisfactory lighting The accident that occurred was fatal The accident occurred at night There will not be a single accident on road if they are satisfactorily illuminated.

LOD-3 Exercise 18.5 (Examination Questions) DIRECTIONS for questions 1 to 5: Below is given a passage followed by several possible inferences which can be drawn from the facts stated in the passage. You have to examine each inference separately in the context of the passage and decide upon its degree of truth or falsity. [IIFT 2005] 1. if the inference is "definitely true", i.e. it properly follows from the statement of facts given 2. if the inference is "probably true" though not "definitely true" in the light of the facts given 3. if the "data is inadequate", i.e. from the facts given you cannot say whether the inference is likely to be true or false. 4. if the inference is "probably false"' though not "definitely false" in the light of the facts given 5. if the inference is "definitely false", i.e. it cannot possibly be drawn from the facts given or its contradicts the given facts. It has been estimated that if current trends continue, by 2025, about ten million people the world over will die each year due to tobacco consumption, doubling the number tobacco-related deaths at present. As of now, half the smoking population of the world will eventually lose their lives to tobacco. Today, tobacco is the fourth most common risk factor for disease and causes the death of one in ten adults worldwide. These statistics are not only a cause for concern in the health sector, but also for the economy of nations. The consumption of tobacco is the highest among the most productive population of a country. Not only does tobacco lead to high public health costs, it also brings down the productivity of users. Whether rich or poor, a tobacco addict spends as much as ten per cent of income on his addiction. The cost of the tobacco habit is devastating for poor families as it cuts into other household expenditures. Despite the statutory warning on cigarette and tobacco pouches, the trend of taking up smoking is continuing. The reasons for this may be many-ranging from peer pressure to persistent advertising to idol imitation, etc. 1. Economies of nations lose much more than what they receive from the tobacco industry. 2. In India, cigarettes are only one form of consumption of tobacco. 3. Smokeless tobacco use is as harmful as smoking. 4. The most susceptible to tobacco advertising is the youth. 5. With the steps already taken for effective tobacco control, Indian tobacco users will find their dreams of easy access to tobacco going up in smoke. Directions Q 6 to 10: Choose that set of statements in which the third statement logically follows from the first two. [CAT 1998] 6. A. No bird is viviparous. D. No bat is a bird. a) ADC B. All mammals are viviparous. C. Bats are viviparous. E. No bird is a mammal. F. All bats are mammals. b) ABE c) FBA d) AFC

7. A. No mother is a nurse. D. Some prude are nurses. a) ABE 8. A. Oranges are fruit. D. Some oranges are sour. a) ADB 9. A. Zens are Marutis. D. All stable are weak. a) ACB

B. Some nurses like to work. C. No woman is a prude. E. Some nurses are women. F. All women like to work. b) CED c) FEB d) BEF B. Some fruits are sour. E. Some oranges are not sour. b) ACE c) BFD B. Zens are fragile. E. Marutis can beat Opels. b) EFD c) CEA C. Oranges are sour. F. Some apples are sour. d) BAC C. Marutis are fragile. F. Opels are stable. d) ABC

10. A. Aardvarks need sleep. B. All animals need sleep. C. Dogs are animals D. Some dogs need sleep. E. Aardvarks are animals. F. Some Aardvarks are dogs. a) BCD b) CEF c) BEA d) BAE DIRECTIONS for questions 11 to 15: Below in each question are given two statements A and B. These statements may be either independent causes or may be effects of independent causes or a common cause. One of these statements may be the effect of the other statement. Read both the statements and decide which of the following answer choices correctly depicts the relationship between these two statements. [IRMA 2005] Mark answer 1, if statement A is the cause and statement B is its effect. Mark answer 2, if statement B is the cause and statement A is its effect. Mark answer 3, if both the statements A and B are independent causes. Mark answer 4, if both the statements A and B are effects of independent causes. Mark answer 5, if both the statements A and B are effects of some common cause. 11. A. The local co-operative credit society has decided to stop giving loans to farmers with immediate effect. B. A large number of credit society members have withdrawn major part of their deposits from the credit society. A. There is sharp decline in the production of oil seeds this year. B. The Govt. has decided to increase the import quantum of edible oil. A. The local farmers approached the NGO seeking advice to protest against the proposed steel plant in the locality. B. The drought situation has worsened for the third consecutive year in the area forcing the local farmers to resort to huge borrowings. A. The local Self Help Group has done a commendable job in improving the quality of life of the poor. B. The rich people in the area have come together and started negative campaign against the Self Help Group. A. All those farmers who are affected by the construction of the thermal power project have decided not to allow the plant to function. B. Majority stakeholders in the thermal power project have decided to go ahead and run the plant.

12. 13.

14.

15.

DIRECTIONS for questions 16 to 20: In each of the questions below one statement is followed by blank spaces which are followed by a conclusion which can be drawn from either the statement given in the question or from the statement given in the question and an additional

statement which may fit in the blank space. You have to study the question and the alternatives which provide a statement and decide which of the answers best fits between the given statement and conclusion. 16. Statement I Blank Space Conclusion A B C Statement II Statement II Statement II : : : Heavy unseasonal rains have lashed most parts of the city causing flood like situation. ___________________________________________________ ___________________________________________________ Govt. has decided to launch a massive food for work program to come out of the crisis. : Almost the entire ripe kharif crop was washed away by the flood water. : Many villages remained marooned for over a week. : Many big cities were inundated and people rambled for food in the tiny shops in the locality.

1. Only A 2. Only B 3. Only C 4. Any one of the above statements will fit. 5. No additional statement is required to draw the conclusion. 17. Statement I Blank Space Conclusion A B C Statement II Statement II Statement II : : : A severe cyclonic storm is likely to hit the western coast by the next fortyeight hours. ___________________________________________________ ___________________________________________________ There was no casualty of human beings and caused considerably less distress to the people living in the western coastline. : The weather department failed to forecast severity of the cyclonic storm. : Most of the people living in the western coast were shifted to safe shelters with food and water. : The Govt. machinery was inadequate to move people out of the coastal area in time.

1. Only A 2. Only B 3. Only C 4. Any one of the above statements will fit. 5. No additional statement is required to draw the conclusion. 18. Statement I Blank Space Conclusion A B C Statement II Statement II Statement II : : : The citizen's awareness forum has agreed to support the Govt. in all the issues related to decreasing environmental pollution. ___________________________________________________ ___________________________________________________ There will be no further violation of norms set up by the environmental pollution control board while setting up industrial units. : In the past, most of the time citizen's awareness group consented setting up industrial units. : Citizen's awareness forum does not have enough resources to monitor all the proposed industrial units. : Citizen's awareness forum has enough influence in the Govt. machinery which sanctions setting up of industrial units.

1. Only A 2. Only B 3. Only C 4. Any one of the above statements will fit. 5. No additional statement is required to draw the conclusion. 19. Statement I Blank Space Conclusion A B C : : : A very large number of people were forced to wade through knee-deep water for long distances as the transport system failed due to waterlogging. ___________________________________________________ ___________________________________________________ The Govt. has instructed all the leading drug manufacturers in the country to release additional quantities of antibiotics in the retail market. : A negligible number of people were found to be suffering from disease caused due to contact of contaminated water. : People having cuts or wounds and travelled through contaminated water are very likely to fall sick immediately after the exposure. : Local doctors have advised their patients to refrain from taking antibiotics without checking with qualified medical practitioners.

Statement II Statement II Statement II 1. Only A 2. Only B 3. Only C 4. Any one of the above statements will fit. 5. No additional statement is required to draw the conclusion. Statement I Blank Space Conclusion : : :

20.

A B C

Statement II Statement II Statement II 1. Only A 2. Only B 3. Only C 4. Any one of the above statements will fit. 5. No additional statement is required to draw the conclusion.

The Govt. has asked all the private telephone service providers to immediately slash down the STD call charges made from the villages with immediate effect. ___________________________________________________ ___________________________________________________ There will be a significant increase in the subscriber base of telephone services in the rural areas. : The private telephone service providers have protested against the government's decision. : People living in the urban areas have also demanded reduction in STD call charges made by them. : People living in the rural areas so far were reluctant to use telephone services.

DIRECTIONS for questions 21 to 25: In each question below is given a statement followed by two courses of action numbered I and II. A course of action is a step or administrative decision to be taken for improvement, follow-up or further action in regard to the problem, policy, etc. On the basis of the information given in the statement, you have to assume everything in the statement to be true, then decide which of the suggested courses of action logically follow(s) for pursuing. Give answer 1, if only I follows. Give answer 2, if only II follows. Give answer 3, if either I or II follows. Give answer 4, if neither I nor II follows. Give answer 5, if both I and II follow.

21.

Statement Courses of action

: Many dilapidated buildings in the city collapsed during monsoon in the city killing many residents. : I. The residents of all the remaining dilapidated buildings should be immediately shifted to other buildings located nearby. II. The residents of all the buildings in bad condition should be served notices to vacate these buildings immediately. : Even after more than fifty eight years of independence, people living below poverty line is still a cause for concern in India. : I. The Government should revamp the public distribution system for providing adequate food to all its citizens. II. The Government should make sincere efforts to generate employment in the rural areas to improve the economic condition of the poor. : People residing in the locality have complained against the highhandedness of the police officers of the local police station in dealing with people approaching the police station with grievances. : I. All the officers of the local police station should immediately be transferred to another police station. II. The administration should immediately investigate into the complaint and take action against erring officers. : Majority of students of class IX of the vernacular medium schools have scored very less marks in Mathematics in the final examination. : I. The school authority should immediately suspend all the Mathematics teachers of the school. II. All these students should be transferred to other English medium schools. : It has been observed that rate of divorce is higher in the cases when both husband and wife are having regular jobs. : I. The working couples should regularly consult the counsellors to save their marriage. II. One member of the couple should be discouraged to take up a regular job.

22.

Statement Courses of action

23.

Statement Courses of action

24.

Statement Courses of action

25.

Statement Courses of action

DIRECTIONS for questions 26 to 30: Below is given a passage followed by several possible inferences which can be drawn from the facts stated in the passage. You have to examine each inference separately in the context of the passage and decide upon its degree of truth or falsity. Mark answer 1, if the inference is 'definitely true', i.e. it properly follows from the statement of facts given. Mark answer 2, if the inference is 'probably true' though not 'definitely true' in the light of the facts given. Mark answer 3, if the 'data are inadequate', i.e. from the facts given you cannot say whether the inference is likely to be true or false. Mark answer 4, if the inference is 'probably false', though not 'definitely false' in the light of the facts given. Mark answer 5, if the inference is 'definitely false', i.e. it cannot possibly be drawn from the facts given or it contradicts the given facts. The cotton textile industry is facing a major shortage of good quality cotton with very little contamination. At present, the weakest link in cotton production was the lack of homegrown

Extra Long Staple (ELS) cotton. This has been the traditional problem with the Indian industry since independence, which has been partially overcome but growing demand is setting the industry back. India produces about 5.3 lakh bales of ELS cotton as against the requirement of 8 lakh bales, so the need of the hour is to see that India's dependence on imported cotton is reduced by improving the quality of domestic ELS cotton. The demand of ELS cotton is likely to increase to 10 lakh bales due to rise in spinning capacity and also due to underpinning of ELS cotton due to its shortage. 26. 27. 28. 29. 30. India regularly imports ELS cotton to meet its requirement. India at present does not have extra spinning capacity. The quality of ELS cotton produced in India is superior than that of other countries. India produces about two-third of its requirement of ELS cotton. There may be more than thirty per cent increase in the demand of ELS cotton in future. Exercise 18.6 (Supporting and Concluding arguments) DIRECTIONS: An argument is a statement meant to convince other person about your point of view. An assertion is a point of view. A counter argument contains logic opposed to your assertion. All the four statements given under each question contains all the four types of arguments given under a, b, c, d below. Based on these definitions, you are required to classify the four sentences in the questions below as: A. concluding assertion B. supporting reason C. statement irrelevant to the argument D. counter argument. 1. a. People treat doctors as messengers of God. b. Doctors of today show more interest in earning money rather than curing patients. c. Medical doctors do a great service to humanity. d. Medical profession is a challenging profession. 1. ADBC 2. BCAD 3. BDAC 4. ACBD a. Films are an important source of entertainment. b. Vulgarity is on increase in films being produced today. c. Films affect ethical standards of society. d. Many films focus on educating the public about social evils. 1. DBAC 2. DABC 3. CBAD

2.

4. CABD

3.

a. Many consumer finance companies provide credit on easy terms to salaried people. b. The purchasing power of middle class people in India is high. c. Indian middle class is still struggling to lead a life of physical comfort. d. Middle class is the largest section in Indian society. 1. BACD 2. BADC 3. ABDC 4. ABCD a. India is a land of spirituality. b. A life of spirituality gives peace of mind. c. In todays India atheism is on the rise. d. During every age great saints have been born in India. 1. ADCB 2. BDCA 3. ACDB

4.

4. BCDA

5.

a. Vedas were written during ancient times. b. People who follow the path shown by Vedas lead a life of fulfillment. c. Vedas are full of wisdom. d. No set of religious writings alone can lead to wisdom for man. 1. CABD 2. CBAD 3. DABC 4. CABD a. Mobile phone leads to accidents as people use it while driving. b. Mobile phones have made communication fast. c. Mobile phone involves advanced technology. d. Today one can speak to anyone in any part of the world at the press of a few buttons. 1. CADB 2. CBDA 3. DBCA 4. DACB a. Boiled water kills germs. b. Without water no living being can exist. c. Even boiled water is not clean if the glass used for drinking is dirty. d. Doctors recommend boiling water to any other form of water purification. 1. ACDB 2. ADCB 3. BDCA 4. BCDA a. Large number of tourists from different countries visit India. b. Indian government is making entry conditions for tourists friendly and easy. c. Tourists enable the economy to develop fast. d. Violence and poor infrastructure deter foreign tourists coming to India. 1. ABCD 2. BADC 3. BACD 4. ABDC a. Industrial development results in more factories, which discharge toxic wastes. b. Developed nations pollute the environment more than developing nations. c. Developed nations have shifted polluting industries to developing nations. d. People in industrialized nation have a higher standard of living. 1. ABCD 2. BADC 3. ABDC 4. BACD a. Women prefer to drive cars, which are safe rather than ride two wheelers. b. Scooters are the most convenient two-wheeler for women to ride. c. Scooters are not as powerful as motorcycles. d. Many women ride scooters in India. 1. DACB 2. CADB 3. CBDA 4. DBCA a. Many monarchs are known to have sacrificed their lives for the protection of the citizens. b. Invariably the monarch rules the country for his selfish gains. c. Monarchy no longer exists in the world. d. Monarchy is not a good form of governance. 1. DBCA 2. CBDA 3. DACB 4. CADB a. Vegetarians are healthier than others. b. The vegetarian diet is such that it is not conducive for diseases. c. Many vegetarians use lot of saturated fat in their diet leading to excessive cholesterol intake. d. Vegetarian food is very tasty. 1. ABCD 2. BACD 3. BADC 4. ABDC a. Some Muslim extremists display behaviour, which is atheistic. b. The term Muslim means believer. c. Muslims are hard working people. d. Muslims are God-fearing people. 1. CADB 2. DACB 3. CBDA

6.

7.

8.

9.

10.

11.

12.

13.

4. DBCA

14.

a. People in developing countries are economically backward. b. India is a developing country. c. In the next 20 years India is likely to become a developed nation. d. Culturally India is far ahead of developed nations. 1.CABD 2. DABC 3. CBAD

4. DBAC

15.

a. The fastest growing sector in India is the service sector. b. Service sector includes several business activities. c. Agriculture is set to grow at a scorching pace. d. Government is giving various incentives for the growth of services in India. 1. BDCA 2. ADCB 3. ACDB 4. BCDA a. Politicians do more harm than good to society. b. Politicians rule the country. c. If politicians do not perform well people vote them out of power. d. A politician works to enhance his personal welfare rather that of his people. 1. ACDB 2. ADCB 3. BDCA 4. BCDA a. Many sports persons develop ailments because of consumptions of steroids. b. Steroids are growth hormones. c. Steroid in limited or appropriate dosage give a boost of energy to the body. d. Anabolic steroid can cause harm to the human body. 1. BDCA 2. BCDA 3. ADCB 4. ACDB a. PSUs generate large number of jobs. b. PSUs tend to be run like government departments without accountability. c. Many private companies are not performing as well as some of the PSUs. d. Privatisation of PSUs will improve their economic performance. 1. DACB 2. DBCA 3. CBDA 4. CADB a. Indian manufacturing companies are facing stiff competition from Chinese firms. b. Chinese companies mostly operate out of China. c. Many Indian Manufacturing firms have outperformed their counterparts across the World. d. The cost structure of Chinese manufacturing firms is very low. 1. ACDB 2. ADCB 3. BDCA 4. BCDA a. Good people help needy persons. b. A beggar is given alms by the good people. c. Peace of mind is the valued possession of good people. d. Many persons who consider themselves to be good do not always show consideration for the poor. 1. BADC 2. ABCD 3. ABDC 4. BACD a. It is observed that competition between many firms in an industry brings down prices of goods and services. b. Firms form a cartel in competitive markets to exploit customers. c. Competition takes place in an industry when two or more players exist. d. Competition results in best products for consumers. 1. ACDB 2. BCDA 3. ADCB 4. BDCA a. Jogging is one form of exercising the body. b. People who jog regularly in the morning live long.

16.

17.

18.

19.

20.

21.

22.

c. Many people prefer walking to jogging as the latter leads to joint pains. d. Jogging is good for health. 1. CBDA 2. DACB 3. DBCA 4. CADB 23. a. Military strength is a necessity in todays context for a country. b. More powerful the defence forces of a country more it is respected by other nations. c. Defence forces are very disciplined in their profession. d. Defence expenditure is a waste as the same money can be used to develop the nation. 1. ABCD 2. ABDC 3. BADC 4. BACD a. Many wealthy persons are philanthropic by nature and donate generously for good causes. b. Rich People are spendthrifts. c. The lifestyle of affluent people is different compared to that of common men. d. Rich people are very calculative when it comes to monetary and business dealings. 1. ABCD 2. ABDC 3. BACD 4. BADC a. Many companies spend lots of money in advertising their products on television. b. Since there are too many advertisements on television people do not clearly remember any one advertisement. c. Television is a major source of entertainment for the average person. d. Television is the best form of media for mass communication. 1. BDCA 2. BCDA 3. ACDB 4. ADCB a. India is the largest producer of quality tea. b. Because of human greed adulterated tea is increasingly being sold in India. c. Tea is the most preferred beverage. d. Darjeeling tea has the best flavour in the world. 1. BCDA 2. ADCB 3. ACDB 4. BDCA a. Executives with business education qualification outperform other executives. b. Business education is passport to a successful career. c. Exponential growth in business education has reduced its quality and job potential. d. Business education is a professional course. 1. ABDC 2. BADC 3. ABCD 4. BACD a. Indian women are very tolerant. b. Modern women in India fight exploitation against them. c. Even when made to do very hard work women in India do not retaliate. d. Indian women are very beautiful. 1. ACBD 2. ADBC 3. BCAD 4. BDAC a. Mass transportation system are safe, economical and fast. b. Competition in passenger car industry is very intense today. c. People are increasingly switching over to car from other modes of transportation. d. Passenger cars are the most convenient mode for commuting in cities. 1. CDBA 2. DCBA 3. CDAB 4. DCAB a. MNCs operate in many countries across several continents. b. There are several Indian companies, which have outperformed MNCs. c. MNCs are more successful than domestic companies in India. d. The market share of MNCs has been increasing during the last decade. 1. DCBA 2. CDBA 3. DCAB 4. CDAB

24.

25.

26.

27.

28.

29.

30.

Answers Exercise 18.1 1. 4 Statement A is fact as it states the time period, B is inference as it states now something is going to happen, C and D are judgments as they give a qualitative assessment. 2. 1 Statement A is inference as it draws a conclusion: we have ceased B and C are facts as they state the situation, and D draws a conclusion, hence it is an inference. 3. 2 A is judgment too well known, B and C are inferences as they draw conclusions, and D is a simple fact. 4. 1 Statement A draws a conclusion, while B is judgment as it mentions a qualitative element meaningful reform, C is an observation, hence fact and D is an inference because of this. 5. 3 Statements A and B are facts as they states facts directly, C is an inference drawn from the facts given, and D is also an inference because of revelations. 6. 5 Statement A is fact as it states directly what they have done, B and D are judgments as they give qualitative opinions, and C is inference If implemented successfully 7. 4 Statement A is fact as it states figures directly, B is judgment as it mentions an opinion indeed large, C is inference as it depends on other actions and D is judgment as it gives advice to newspapers. 8. 1 Statement A and B are inferences drawn from other facts, C gives an opinion hence judgment, while D is again inference as a result 9. 2 Statement A and B are facts as they state direct facts, C is judgment as it expresses opinion: touted as, while D is inference drawn from other facts. 10. 4 Statement A and B are facts as they state symptoms directly, C and D are inferences if administered, and high on the drug. 11. 3 Statement A is fact as it states a direct relationship, B is judgment as it mentions a desirable objective, while C and D are direct facts. 12. 5 Statement A is judgment as it states important step, B, C and D are inferences from other facts. 13. 1 Statement A is inference from research, B is a direct fact, C is inference as it says it is likely and D is judgment: most authoritative 14. 3 Statement A is inference from watching Tony Blair, all other statements are judgments: B: appropriate description, C: its time to and D: will have a lot more clout 15. 1 Statement A is inference, in exercising one right, B: When a political party, C is inference from their economic conditions. D is judgment as it says that they have no effect 16. 4 Statement A is inference from the property market, B is judgment: beyond the reach of buyers C is fact, and D is inference from derivatives market. 17. 2 Statements A and B are inferences, note the words reflect and therefore; C is judgment because of humiliation, and D is fact as it states what he believes in. 18. 4 Statement A is inference from growth data, B and C are direct facts, D is again inference from data. 19. 5 Statement A, B and C are facts as they are his statements, D is inference from the data that other people are not convinced. 20. 1 Statement A is direct fact as it says what workers did, B is inference as it is thought to contain, C and D are facts explaining what the experiments did. 21. 3 Statement A is inference after the blasts, B is judgment as it describes a predicament, C is inference as it shows course of action, and D is a direct fact. 22. 2 Statement A is inference as it comes from mindset, B is judgment as it gives advice, C is inference from the letter, and D is judgment as it gives an opinion about it being unlikely. 23. 1 Statements A and B are inferences as they are based on conditions, C is a judgment we need to and D is a straight forward fact. 24. 4 Statement A is inference as it is based on another fact, B and C are facts as they address the statement directly, and D is an inference based on the above statements.

25. 3 26. 1 27. 5 28. 2 29. 2 30. 1

Statement A is judgment as it asks a question taking the answer for granted, B is an inference as it gives a reason, C and D are again judgments as they make value judgments. Statement A is judgement as it mentions a need, which is subjective, B and C are inferences as they are drawn from other facts, and D is a direct fact. Statements A and B are inferences as they draw conclusions, while C and D are judgments since they make opinion based statements. Statement A is inference as it draws from another fact, B is judgement they benefit nobody C is a fact and D is inference drawing upon another action. All the statements have an element of judgment in them and reflect the authors opinions. All the statements have an element of judgment in them and reflect the authors opinions.

Exercise 18.2 1. 2 If D is not true, then A must be true. 2. 1 A causes B; If B happens, A will also not happen. 3. 4 4. 1 Horror story causes nightmare. If I did not have nightmare, then its cause would not have happened. 5. 2 Same as above. 6. 1 7. 5 8. 5 9. 3 10. 2 11. 4 12. 2 13. 4 14. 5 15. 2 16. 2 17. 1 18. 5 19. 3 20. 2 21. 2 22. 2 23. 2 24. 3 25. 2 26. 3 27. 2 28. 4 29. 2 30. 4 Exercise 18.3 1. 4 The words are synonyms 2. 1 Second word is extreme form of the first. 3. 1 The words are antonyms. 4. 2 The first causes the second. 5. 2 Pride is a group of lions. Pride stays in a forest. Colony is a group of houses. Colony lies in a city. 6. 2 Common to Poet & Prophet vision; Common to killer & Terrorist Violence 7. 3 The words are opposites. 8. 1 The first word describes the second.

9. 4 10. 4 11. 4 12. 2 13. 1 14. 1 15. 2 16. 2 17. 3 18. 2 19. 4 20. 3 21. 1 22. 4 23. 3 24. 1 25. 3 26. 2 27. 2 28. 4 29. 2 30. 1

The words are opposites. The second word is a consequence of the first. The second word refers to a cheap form of the work produced by the poet or writer. The second word refers to small quantities of the first word. The first refers to higher degree of the second word. The first word refers to alarm or warning for the second. The second word refers to the consequence of the first word. The other choices are synonyms but the second choice is opposites. All the other choices are opposites. The other choices are synonyms. The other choices define the set to which the animal belongs, vulpine if fox like. Preamble is introductory to Constitution, prologue is introductory to play. Horses neigh, donkeys bray. Meter is measure of distance, kilogram is measure of weight. Always is opposite of never, all is opposite of none. Umbrella protects from rain, dam from flood. Conscience is defining quality of pacifist, religion of a sacred person. Shedding of feathers by a bird is moulting. Pound is currency of England, lira of Italy. Toronto is capital of Canada, Rangoon of Burma. Each refers to single quantity, as part to whole.

Exercise 18.4 1. 2 Argument I is against the truth. Computerisation no longer demands a lot of money. Argument II makes sense. If we have to compete with rest of the world, we have follow the new trends and technologies. 2. 2 Personal tax doesnt stop people from earning more. Hence argument I is weak. Argument Ii is strong. A social welfare economy demands sharing of wealth. 3. 3 Either of the two arguments can be strong. 4. 1 5. 3 There is no information about the situation before reforms. 6. 2 7. 1 8. 4 9. 1 All C belongs to F and some F is also C. 10. 2 Either M and D intersect or they do not, hence either II or IV. 11. 3 If we use detergents to clean, then we can say that they dislodge dirt. 12. 4 We can assume that people leave because of better conditions, but not its effect. 13. 3 14. 1 15. 3 16. 2 17. 2 18. 1 19. 4 20. 2 21. 3 If we know Manoj is right, then his statement is also right. 22. 4 23. 1 24. 3 25. 4 We cant say any of the given statements are facts. Since all metros have ring roads and Delhi is metro, hence it has a ring road. We can conclude that some musicians are trained for 10 years from Fact 2. The jeep can be parked, at any of the three possible locations, X, Y or Z. In every possible case, statement in option 4th, holds true.

26. 5 27. 2 28. 3 29. 1 30. 2 Exercise 18.5 1. 2 Since no figures are given, we cannot say it is definitely true, but the argument does point to loss of economies. 2. 3 No information is given about forms of tobacco. 3. 3 No information is given about smokeless tobacco. 4. 2 It talks about the most productive population, which could include youth. It is not definitely true as this could include young adults and youth are not productive per se. 5. 3 No information is given about tobacco control in India. 6. 2 7. 3 8. 1 9. 1 10. 3 11. 2 B is the reason why the society is not giving out loans. 12. 1 A is the reason that oil has to be imported as there is a shortage of oil seeds. 13. 3 The statements are unrelated. 14. 1 Because the self help group has been successful, there has been a campaign against it. 15. 5 Both actions are a result of a common cause of having a thermal power plant. 16. 1 Only A links up the governments decision with the washing away of the crop. The others do not match with the conclusion. 17. 2 As the damage was less, it means that the steps taken were successful in helping the villagers. 18. 3 As it has influence, it matches with the conclusion of no further violation. The other statements do not match. 19. 2 The reason for the government action is that it fears that people will suffer from infections and more antibiotics should be released. 20. 5 None of the statements are required to reach the conclusion of the subscriber base being widened. 21. 3 Either the residents should be shifted or given notices to shift out. 22. 5 Both actions are necessary to resolve the question of people living below the poverty line. 23. 2 Transferring officers is not a solution, hence (1) is wrong. Taking action will certainly resolve the problem. 24. 4 Neither of the given solutions will help solve the problem of students failing in the exam. 25. 1 The solution is given in the first course of action. The second is not a solution to the problem. 26. 1 It is clearly mentioned that India's dependence on imported cotton should be reduced, which means that it regularly imports ELS cotton. 27. 2 The statement is probably true because it is mentioned that spinning capacity is rising. This means that there might be a shortage of capacity, that is why it is increasing. 28. 3 No data is given about the quality of cotton in India compared to the ELS cotton in other countries. 29. 1 It is clearly mentioned that India produces 5.3 lakh bales of ELS cotton against requirement of 8 lakh bales, which is about two-thirds. 30. 5 The increase in demand is expected to be 10 lakh bales against the present demand of 8 lakh bales, which is 25%, hence the statement is false. Exercise 18.6

1. 3 2. 3 3. 2 4. 3 5. 4 6. 3 7. 4 8. 1 9. 2 10. 1 11. 2 12. 4 13. 4 14. 4 15. 2 16. 2 17. 1 18. 3 19. 1 20. 2 21. 4 22. 1 23. 1 24. 3 25. 4 26. 2 27. 1 28. 2 29. 2 30. 4

The second statement is countering the idea of service while the last statement is irrelevant. The example of vulgarity is the supporting reason, while the first statement is irrelevant. The first is an example to support, the second becomes the main argument, while the third is countering. (a) is inferred from (d) while (c) is a counter argument. (a) is irrelevant, while (b) and (c) are related. The first counters the idea, while (b) and (d) are related. (d) is the concluding argument and (a) supports it. (b) supports (a) while the last goes against the idea. (c) counters the idea while (d) is irrelevant. The first statement is about cars, hence counters the argument while (b) is concluding. (d) is the concluding statement while (b) supports it. (a) is the concluding argument while (b) supports it. (d) is concluding while the first is irrelevant. (c) is concluding. (d) supports (a). (d) supports (a). (a) is the conclusion while (d) is supporting it. (b) supports (d) (d) supports (a) while (b) is irrelevant. (b) supports (a) while (d) counters the argument. (a) supports (d) while (c) is irrelevant. (b) supports (a) while the first statement is irrelevant. (b) supports (a) while the last is a counter argument. (b) supports (a) while the last is a counter argument. (d) supports (a) while (c) is irrelevant. (a) is the conclusion because (d) supports it, (c) is irrelevant. (b) supports (a) while (d) is irrelevant. The first statement is the conclusion, while the last is irrelevant. (d) is the concluding statement. (c) is the conclusion and (d) is the supporting reason.

Das könnte Ihnen auch gefallen